Saltar al contenido principal
LibreTexts Español

15.5: Tercera Ecuación de Maxwell

  • Page ID
    131907
  • \( \newcommand{\vecs}[1]{\overset { \scriptstyle \rightharpoonup} {\mathbf{#1}} } \)

    \( \newcommand{\vecd}[1]{\overset{-\!-\!\rightharpoonup}{\vphantom{a}\smash {#1}}} \)

    \( \newcommand{\id}{\mathrm{id}}\) \( \newcommand{\Span}{\mathrm{span}}\)

    ( \newcommand{\kernel}{\mathrm{null}\,}\) \( \newcommand{\range}{\mathrm{range}\,}\)

    \( \newcommand{\RealPart}{\mathrm{Re}}\) \( \newcommand{\ImaginaryPart}{\mathrm{Im}}\)

    \( \newcommand{\Argument}{\mathrm{Arg}}\) \( \newcommand{\norm}[1]{\| #1 \|}\)

    \( \newcommand{\inner}[2]{\langle #1, #2 \rangle}\)

    \( \newcommand{\Span}{\mathrm{span}}\)

    \( \newcommand{\id}{\mathrm{id}}\)

    \( \newcommand{\Span}{\mathrm{span}}\)

    \( \newcommand{\kernel}{\mathrm{null}\,}\)

    \( \newcommand{\range}{\mathrm{range}\,}\)

    \( \newcommand{\RealPart}{\mathrm{Re}}\)

    \( \newcommand{\ImaginaryPart}{\mathrm{Im}}\)

    \( \newcommand{\Argument}{\mathrm{Arg}}\)

    \( \newcommand{\norm}[1]{\| #1 \|}\)

    \( \newcommand{\inner}[2]{\langle #1, #2 \rangle}\)

    \( \newcommand{\Span}{\mathrm{span}}\) \( \newcommand{\AA}{\unicode[.8,0]{x212B}}\)

    \( \newcommand{\vectorA}[1]{\vec{#1}}      % arrow\)

    \( \newcommand{\vectorAt}[1]{\vec{\text{#1}}}      % arrow\)

    \( \newcommand{\vectorB}[1]{\overset { \scriptstyle \rightharpoonup} {\mathbf{#1}} } \)

    \( \newcommand{\vectorC}[1]{\textbf{#1}} \)

    \( \newcommand{\vectorD}[1]{\overrightarrow{#1}} \)

    \( \newcommand{\vectorDt}[1]{\overrightarrow{\text{#1}}} \)

    \( \newcommand{\vectE}[1]{\overset{-\!-\!\rightharpoonup}{\vphantom{a}\smash{\mathbf {#1}}}} \)

    \( \newcommand{\vecs}[1]{\overset { \scriptstyle \rightharpoonup} {\mathbf{#1}} } \)

    \( \newcommand{\vecd}[1]{\overset{-\!-\!\rightharpoonup}{\vphantom{a}\smash {#1}}} \)

    Esto se deriva del teorema de Ampère, que es que la línea integral del campo magnético\(\textbf{H}\) ar alrededor de un circuito cerrado es igual a la corriente encerrada.

    Ahora hay dos componentes posibles a la corriente “encerrada”, uno de los cuales es obvio, y el otro, supongo, también podría decirse que es “obvio” una vez que se ha señalado! Tratemos primero con el inmediatamente obvio, y veamos la Figura XV.1.

    FIGURA XV.1

    En la Figura XV.1, me imagino un cilindro metálico con corriente que fluye de arriba a abajo, es decir, electrones que fluyen de abajo hacia arriba. Sin embargo, no tiene por qué ser un cilindro metálico. Podría ser simplemente un volumen de espacio con una corriente de protones moviéndose de arriba a abajo. En cualquier caso, la densidad de corriente (que puede variar con la distancia desde el eje del cilindro) es\(\textbf{J}\) , y la corriente total encerrada por el círculo discontinuo es la integral de\(\textbf{J}\) todo el cilindro. En una geometría más general, en la que no\(\textbf{J}\) es necesariamente perpendicular al área de interés, y de hecho en la que el área no necesita ser plana, ésta sería\(\int \textbf{J} \cdot \textbf{d}\boldsymbol{\sigma}\).

    Ahora para el componente menos obvio a la “corriente encerrada”. Ver Figura XV.2.

    FIGURA XV.2

    En la Figura XV.2, imagino dos placas de condensador en proceso de carga. Sin duda hay una corriente que fluye en los cables de conexión. Hay un campo magnético en A, y la línea integral del campo alrededor de la curva punteada superior es indudablemente igual a la corriente encerrada. La corriente es igual a la velocidad a la que se está acumulando carga en las placas. Los electrones están siendo depositados en la placa inferior y están saliendo de la placa superior. También hay un campo magnético en B (¡no se detiene de repente!) , y el campo en\(B\) is just the same as the field at A, que es igual a la velocidad a la que se está acumulando la carga sobre las placas. La carga en las placas (que puede no ser uniforme, y de hecho no lo será mientras la corriente sigue fluyendo o si las placas no son infinitas en extensión) es igual a la integral de la densidad de carga multiplicada por el área. Y la densidad de carga en las placas, según el teorema de Gauss, es igual al campo eléctrico\(\textbf{D}\) entre las placas. Así la corriente es igual a la integral de\(\dot{ \textbf{D}}\) sobre la superficie de las placas. Por lo tanto, la integral de línea\(\textbf{H}\) alrededor de cualquiera de los bucles cerrados discontinua es igual a\(\int \dot{\textbf{D}} \cdot \textbf{d} \boldsymbol{\sigma}\).

    En general, ambos tipos de corriente (la obvia en la que hay un flujo obvio de carga, y la menos obvia, donde el campo eléctrico varía debido a un flujo real de carga en otra parte) contribuyen al campo magnético, por lo que el teorema de Ampère en general debe leer

    \[ \int_{loop} \textbf{H} \cdot \textbf{ds} = \int_{area} (\dot{ \textbf{D}} + \textbf{J}) \cdot \textbf{d}\boldsymbol{\sigma} \tag{15.5.1} \label{15.5.1}\]

    Pero la integral de línea de un campo vectorial alrededor de una curva plana cerrada es igual a la integral de la superficie de su rizo, y por lo tanto

    \[ \int_{area} \text{curl} \, \textbf{H} \cdot d{\sigma} = \int_{area} (\dot{ \textbf{D}} + \textbf{J}) \cdot \textbf{d} \boldsymbol{\sigma} \tag{15.5.2} \label{15.5.2}\]

    Así llegamos a:

    \[\text{curl}\, \textbf{H} = \dot{ \textbf{D}} + \textbf{J}\tag{15.5.3} \label{15.5.3}\]

    o, en la notación nabla,

    \[\boldsymbol{\nabla} \times \textbf{H} = \dot {\textbf{D}} + \textbf{J}\tag{15.5.4} \label{15.5.4}\]

    Esta es la tercera de las ecuaciones de Maxwell.


    This page titled 15.5: Tercera Ecuación de Maxwell is shared under a CC BY-NC 4.0 license and was authored, remixed, and/or curated by Jeremy Tatum via source content that was edited to the style and standards of the LibreTexts platform; a detailed edit history is available upon request.